Inverse transformation for linear transformations

Click For Summary
The discussion focuses on finding the inverse transformations for given linear transformations T(s, t). Participants express confusion about the concept of inverse transformations, particularly in the context of multiple variables. It is clarified that a transformation maps vectors from one space to another, and the inverse transformation should map back to the original vector. For linear transformations, a matrix can represent the transformation, while nonlinear transformations require a different approach to describe their inverses. Understanding the nature of each transformation is crucial for determining the appropriate method for finding the inverse.
SteelDirigibl
Messages
40
Reaction score
0

Homework Statement


Find the inverse transformation T-1(x, y) in each case below:

(a) T(s,t) = (3s+t, s-2t)
(b) T(s,t) = (s3, s+t)
(c) T(s,t) = (t, s)
(d) T(s,t) = (et, s)



The Attempt at a Solution


I'm not completely sure what this is asking. What is an inverse transformation? I'm sure it's really basic, I just can't find an explanation online. The only examples I can find have one variable. So I can't really get anywhere on this.
 
Physics news on Phys.org
A transformation is a kind of function. Instead of mapping one number to another as ordinary functions do, it maps a vector to some other vector, possibly in a different vector space.

For example, if s = 2 and t = 1, the transformation in part a maps the vector <2, 1> to the vector <7, 0>. The inverse transformation, T-1 would map <7, 0> back to <2, 1>.

If a transformation is linear (as a and c are), there is a matrix that represents the transformation. The ones in b and d are nonlinear, so "finding the inverse transformation" might involve only describing what the inverse transformation needs to do.
 
Last edited:
Question: A clock's minute hand has length 4 and its hour hand has length 3. What is the distance between the tips at the moment when it is increasing most rapidly?(Putnam Exam Question) Answer: Making assumption that both the hands moves at constant angular velocities, the answer is ## \sqrt{7} .## But don't you think this assumption is somewhat doubtful and wrong?

Similar threads

  • · Replies 10 ·
Replies
10
Views
2K
  • · Replies 1 ·
Replies
1
Views
1K
Replies
2
Views
2K
  • · Replies 4 ·
Replies
4
Views
3K
  • · Replies 31 ·
2
Replies
31
Views
4K
  • · Replies 3 ·
Replies
3
Views
3K
  • · Replies 11 ·
Replies
11
Views
2K
  • · Replies 6 ·
Replies
6
Views
3K
  • · Replies 8 ·
Replies
8
Views
2K
  • · Replies 8 ·
Replies
8
Views
2K